LSAT and Law School Admissions Forum

Get expert LSAT preparation and law school admissions advice from PowerScore Test Preparation.

 sdlee4
  • Posts: 5
  • Joined: Sep 27, 2012
|
#5775
Hi, I understand why answer choice (A) may be correct, but could you explain why answer choices B-D would be incorrect.

Thanks,
Sam
 Steve Stein
PowerScore Staff
  • PowerScore Staff
  • Posts: 1153
  • Joined: Apr 11, 2011
|
#5795
In that one, two groups are compared, one on a low-carb (and high protein) diet and the other on a high carb (and low protein) diet. The low carb dieters lost more weight, so the author concludes that a low carb diet with high protein is the best way to lose body fat.

The correct answer choice will be the one that shows either that the high protein diet is not as good as it seemed, or that the low-protein diet compares better than it seems based on the information in the stimulus.

Correct answer choice A provides the the low protein diet also yields body fat losses, meaning that the low protein diet is better than it seemed with regard to lowering body fat.

Was there an incorrect answer choice that you found particularly appealing? Let me know--thanks!

~Steve
 sdlee4
  • Posts: 5
  • Joined: Sep 27, 2012
|
#5803
O okay, yea I guess I could see why A is correct because this answer choice is the only one that at least weakens the argument by casting doubt on the conclusion and offering the possibility that the low-carb diet is NOT the MOST effective diet to lose body fat.

I guess this is another example of a tougher LSAT question in which the correct answer doesn't necessarily destroy the argument, but does weaken it.

Thanks
 eober
  • Posts: 107
  • Joined: Jul 24, 2014
|
#16414
Hi,

Concerning answer choice A, even if water compensates for the fat lost isn't it still the case that body fat of the person is reduced, it is only compensated by water (but not fat). Could you clarify this answer choice and how it weakens the argument?
 David Boyle
PowerScore Staff
  • PowerScore Staff
  • Posts: 836
  • Joined: Jun 07, 2013
|
#16436
eober wrote:Hi,

Concerning answer choice A, even if water compensates for the fat lost isn't it still the case that body fat of the person is reduced, it is only compensated by water (but not fat). Could you clarify this answer choice and how it weakens the argument?
Hello,

The argument is that high protein is better since the weight loss proves fat-burning. Answer A hurts that, since it says that low protein causes water retention, which shows that even if there isn't weight loss, there can still be fat-burning.

David
 Sherry001
  • Posts: 81
  • Joined: Aug 18, 2014
|
#20323
Hello hello;
Could you please walk me through this question? I suffered . I just cannot wrap my head around this question. here is my how I approached this question.


1- After 10 days those on the low card diet had lost MORE WEIGHT than those on the high diet.
C: The most effective way to lose BODY FAT is to eat lots of protein and no carbs.


My thinking before looking at the answers: Since it’s a weaken question I need to either attack the conclusion or show that maybe there is a negative side to taking low carbs. Maybe in the long run it would lead to obesity or health problems ( maybe then it wont be the most effective way. )
Then after going through every single answer choice and not liking anything and getting the question wrong , I realized well one can lose body fat without losing weight. Maybe I used to weigh 115LB, and lost some fat after the diet, but now I tuned my fat into muscle and I weight 300LB.

A) essentially this is telling me that in the study, those that had the high carbs lost body fat, where as the other group with the low carbs did not lose body fat even if they lost weight? ( I cant wrap my head around this. I could accept if they lost body fat but not weight, but could they lose body weight and not fat)? SUPER LOST!

B) This is what I had picked . I saw this as weakening the conclusion a bit, since the new regimen required low carb and high protein , so if this leads to weight gain I thought it would make the author question his conclusion. OR is this wrong because It makes it all that more interesting that high protein would lead to significant weight gain and yet in the authors group it led to the opposite effect? SUPER LOST!

C)This is saying the high protein and low carb will not allow the body to turn fat into muscle, without losing significant weight lost. So maybe this is irrelevant, if not strengthening the weight lost occurring due to low carb.

D)This strengthens the low carb diet causing weight lost.

E)Also strengthens the idea of the low carb diet, by indicating cause doesn’t occur effect doesn’t occur.

Thank you so much !
Sherry
 Steve Stein
PowerScore Staff
  • PowerScore Staff
  • Posts: 1153
  • Joined: Apr 11, 2011
|
#20346
Hey Sherry,

That's a good question. Part of the challenge in this case is dealing with the constant back-and-forth between highs and lows. To keep things simple, lets call the first group the high protein team, and the second group the low protein team.

After ten days, the high protein team had lost more weight, and the author jumps to the conclusion that the best way to lose body fat is to be on a high protein (and low carb) diet.

Since a Weaken question follows, the correct answer choice might show that there are better ways to lose body fat, or perhaps that the results of the study were somehow misleading...

Answer choice (A) provides that a low-protein diet can cause water retention, but a high-protein diet does not. This would mean that it is possible that the low-protein team actually lost more fat than the other team—water retention could have hidden that fact by driving up the (water) weight of the low-protein team—and making it appear that members of that team were in worse condition than they actually were.

Answer choice (B) doesn't provide much information since the term "many" is so vague. Further, the author's conclusion refers to a diet high in protein and low in carbs.The "many" people discussed in this choice may all have consumed large quantities of protein and large quantities of carbohydrates, in which case their example would not weaken the author's conclusion at all.

I hope that's helpful! Please let me know whether this is clear—thanks!

~Steve
 laurat
  • Posts: 8
  • Joined: Apr 03, 2017
|
#34449
I was wavering between A and B and chose B. My prephrase anticipated that losing weight doesn't necessarily mean that body fat is lessened, which I thought B outlined perfectly. A to me seems to question the study but doesn't seem nearly as strong as B. I was positive I had gotten this question right-can you help?

(PT-52-61 p 138)
 AthenaDalton
PowerScore Staff
  • PowerScore Staff
  • Posts: 296
  • Joined: May 02, 2017
|
#34482
laurat wrote:I was wavering between A and B and chose B. My prephrase anticipated that losing weight doesn't necessarily mean that body fat is lessened, which I thought B outlined perfectly. A to me seems to question the study but doesn't seem nearly as strong as B. I was positive I had gotten this question right-can you help?

(PT-52-61 p 138)
Here's one way to distinguish between (A) and (B):

The study described in the prompt that led to weight loss involved a diet that was BOTH high in protein AND low in carbs. But (B) only talks about eating high amounts of protein, nothing about carbs. (B) could presumably refer to people who eat lots of carbs AND lots of protein -- it wouldn't be much of a surprise if that group of folks gained body fat. In any case, it does nothing to weaken the study's conclusion that eating lots of protein (and few carbs) leads to a loss in body fat.

By contrast, (A) does weaken the study's conclusion because it (1) refers directly to one of the groups in the study (low-protein; high-carbs) and (2) provides a reason why Group A could have lost more pounds, but Group B could have lost more body fat.

So, to be clear, what's referred to in (A) could (hypothetically) look like this:

Group A: lost an average of 20 pounds, consisting of 10 pounds lost water and 10 pounds lost body fat.
Group B: lost an average of 15 pounds, consisting of 15 pounds lost body fat.

Under this (hypothetical) study result, which is what is described in (A), Group B lost more bodyfat even though they lost less weight overall.

Hope this makes sense -- good luck!

Athena Dalton
 lsacgals101
  • Posts: 28
  • Joined: Mar 31, 2019
|
#65664
Why is answer C incorrect?

Get the most out of your LSAT Prep Plus subscription.

Analyze and track your performance with our Testing and Analytics Package.